Which is stronger? 3 scoops of coffee for 4 cups of water, or 4 scoops of coffee for 6 cups of water? Show Your Work.

Answers

Answer 1

The stronger one is 4 scoops of coffee for 6 cups of water.

How strong coffee is made?

Strong coffee is prepared by employing a greater coffee grounds-to-water ratio. Typically, 2 teaspoons of coffee grinds are used for every 6 ounces of water in a strong cup of coffee. To make a truly strong cup of coffee, use 3 teaspoons of ground coffee for every 6 ounces of water. If you use a French press, you need also increase the steeping time.

In the given question, it is stronger to use 4 scoops of coffee for 6 glasses of water. This is due to the fact that there are more coffee grounds in relation to the amount of water, resulting in a higher concentration of coffee in the combination.

To know more about coffee, visit

brainly.com/question/29758432

#SPJ1


Related Questions

6. Application Juanita, who is 1.82 meters tall, wants to
find the height of a tree in her backyard. From the
tree's base, she walks 12.20 meters along the tree's
shadow to a position where the end of her shadow
exactly overlaps the end of the tree's shadow. She is
now 6.10 meters from the end of the shadows. How
tall is the tree?

Answers

The height of tree is 9.9372.

What is the congruent triangle?

Two triangles are said to be congruent if the length of the sides is equal, a measure of the angles are equal and they can be superimposed.

ΔABC ~  ΔDEF  only if ratio of two sides of ΔABC and corresponding two sides of ΔDEF is equal and the angle included in both sides are congruent.

Given;

Juanita is 1.82 meters tall

From the tree's base, she walks 12.20 meters along the tree's shadow to a position where the end of her shadow.

She is now 6.10 meters from the end of the shadows.

Now,

Let, h be the height of the tree;

Here, ΔABC ~ ΔADE,

So, DE/BC = AD/AB

h/1.82 = (6.10 + 12.20)/6.10

h/1.82= 18.30/6.10

h=33.306/6.10

h=5.46

h = 1.82*5.46

= 9.9372

Therefore, by congruency of triangle the answer will be 9.9372.

Learn more about congruent triangles;

https://brainly.com/question/12413243

#SPJ1

someone help me please and thank you

Answers

Ordinal data can be ordered, as the naming implies. Both have "ord" to help remember the connection. An example of this would be something like small, medium, large. The items do not have to be numeric in nature.

Nominal data is any category or name. Order isn't present here. For example, if a survey asked about a person's favorite color, then there isn't any inherent order here.

Find GI, AGI, TI...
Given:
Wages : $145,000
Interest : $2500,
Contribution towards the retirement: $3000
Exemptions: $4100
Deductions: $2800

-Thanks!

Answers

The GI (Gross Income) is $147,500, AGI (Adjusted Gross Income) is $144,500, and TI (Taxable Income) is $137,600.

What is Interest?

Interest is the income that is returned to the holder of capital. Different kinds of interest include gross interest and net interest.

To find GI (Gross Income), we add wages and interest:

GI = Wages + Interest = $145,000 + $2500 = $147,500

To find AGI (Adjusted Gross Income), we subtract the contributions towards retirement from GI:

AGI = GI - Contributions towards Retirement = $147,500 - $3000 = $144,500

To find TI (Taxable Income), we subtract exemptions and deductions from AGI:

TI = AGI - Exemptions - Deductions = $144,500 - $4100 - $2800 = $137,600.

Therefore, the taxable income is $137,600.

Hence, GI (Gross Income) is $147,500, AGI (Adjusted Gross Income) is $144,500, and TI (Taxable Income) is $137,600.

To learn more about the Interest visit:

https://brainly.com/question/30393144

#SPJ1

The PTO has 1,932 tickets to sell for the school carnival. They want to split the tickets equally into at least 3, but less than 10 ticket booths. How many ticket booths could they split them into without having any left over? Please explain your answer.​

Answers

The PTO could split the 1,932 tickets into either 3, 6, 7, 8, or 9 ticket booths, without having any tickets left over.

How  many ticket booths could they split?

We need to find a number that is divisible by at least 3, but less than 10.

We can start by listing out the multiples of 3 and see which one is closest to 1,932:

3 × 1 = 3

3 × 2 = 6

3 × 3 = 9

3 × 4 = 12

3 × 5 = 15

3 × 6 = 18

3 × 7 = 21

3 × 8 = 24

3 × 9 = 27

We can see that the closest multiple of 3 to 1,932 is 1,932 ÷ 3 = 644. This means that if the PTO split the tickets into 644-ticket batches, they would have 3 equal booths of 644 tickets each.

We can check whether 644 is also divisible by 4, 5, 6, 7, 8, or 9, as those numbers are less than 10. We can see that 644 is not divisible by 4 or 5, but it is divisible by 6, 7, 8, and 9. This means that the PTO could also split the tickets into 6, 7, 8, or 9 ticket booths, with each booth having an equal number of tickets.

Therefore, the PTO could split the 1,932 tickets into either 3, 6, 7, 8, or 9 ticket booths.

Learn more about number of tickets here:https://brainly.com/question/9512634

#SPJ1

Can someone pls help me?
What is the initial value of the function?

Answers

The initial value of the function is 1.

What is a function?

A function in mathematics from a set X to a set Y assigns exactly one element of Y to each element of X. The sets X and Y are collectively referred to as the function's domain and codomain, respectively. Initially, functions represented the idealized relationship between two varying quantities.

A relationship in which one input value exactly equals one output value is known as a function. A function's initial value is a crucial component. A starting value or starting point is exactly what it sounds like: It is an initial value.

The line intersects the y-axis at (0,1).

The initial value of the function is 1.

To learn more about rate of change of a function, click on the below link:

https://brainly.com/question/1611478

#SPJ1

Marvin was 1.55m tall a year ago. now,he is 1.62m tall. find the percentage increase in Marvin's height correct to 2 decimal places​

Answers

Answer:

4.52 %

Step-by-step explanation:

the percentage increase in height is calculated as

[tex]\frac{increase}{original}[/tex] × 100%

increase in height = 1.62 - 1.55 = 0.07 ,

then

% increase = [tex]\frac{0.07}{1.55}[/tex] × 100% = 0.045.. × 100% ≈ 4.52% ( to 2 decimal places )

Color the rational numbers blue
Leave the irrational numbers alone

Answers

Answer:

The answer is provided in the image below.

write the sentence as an equation

n and 328 more is the same as 13

Answers

The equation which correctly represents the sentence in discuss as required is; n + 328 = 13.

Which sentence correctly represents the equation?

It follows from the task content that the equation which correctly represents the sentence given in the task content is to be determined.

The word phrase; n and 328 more can be represented as; n + 328.

Therefore, n and 328 more is the same as 13 can be represented as; n + 328 = 13.

Read more on word phrase to algebraic expressions;

https://brainly.com/question/25991924

#SPJ1

Data analysts pay attention to sample size in order to achieve what goals? select all that apply.

Answers

The practise of extracting information from data to guide better business decisions is known as data analysis. Five iterative phases typically comprise the data analysis process:

Choose the data you want to examine.

gather the data

Clean up the data before analysis.

Study the information

Interpret the analysis' findings

It is common knowledge that data analysts focus on the sample size in order to

To prevent a few outlier responses from distorting the results

ensuring that the data reflects a variety of viewpoints

to prevent making erroneous conclusions due to a tiny sample size

An analyst of data is who?

An individual involved in the handling of data is referred to by this phrase.

Such a person aids in the analysis, retrieval, and drawing of conclusions from the information at hand.

know more about data analysts click here;

https://brainly.com/question/23810306

#SPJ4

Chris buys a new car for $24000, if he pays $1200 in taxes, what is the effective tax rate?

Answers

Answer:

Step-by-step explanation:

The effective tax rate is the amount of tax paid as a percentage of the total purchase price. To calculate the effective tax rate, you can divide the amount of tax by the total purchase price and multiply the result by 100 to get the answer as a percentage.

In this case, the effective tax rate would be:

(1200 / 24000) * 100 = 5%

So Chris's effective tax rate on the purchase of his car is 5%.

The area of Rhode Island is about 1000
square miles. The area of Alaska is about
5.7 x 105 square miles.
Complete the statement based on the
information above:
Alaska's area is ___ times the area of rhode island

Answers

Answer:

57

Step-by-step explanation:

To find the ratio of the area of Alaska to the area of Rhode Island, we need to divide the area of Alaska by the area of Rhode Island.

The area of Rhode Island is 1000 square miles and the area of Alaska is 5.7 x 10^5 square miles.

So, the ratio is: 5.7 x 10^5 / 1000 = 57

Therefore, Alaska's area is 57 times the area of Rhode Island.

5.7 x 10^5 divided by 1000 = 57.

Match each polynomial on the left with its two factors on the right.
x³-27
x² +27





x+3
x-3
x² + 3x +9
x² + 3x-9
x²-3x+9
x²-3x-9

Answers

Answer:

hi

Step-by-step explanation:

[tex]x {}^{3} - 27 = {x}^{3} - {3}^{3} \\ = (x - 3)( {x}^{2} + 3x + 9) \\ {x}^{3} + 27 = {x}^{3} + {3}^{3} \\ = (x + 3)( {x}^{2} - 3x + 9)[/tex]

The ratio of horizontal distance to height of the ramp is 15:1. A builder has a roll of nonslip rubber mat that is 15 feet long. Does he have enough forever to cover the Ramp completely?

Answers

Answer:

No. The length of the ramp is [tex]\sqrt{226[/tex] The rubber mat will be too short

Step-by-step explanation:

Isosceles trapezoid KRWT is shown

Given KRWT is an isosceles trapezoid with KR and TW
Prove: WK = TR

An incomplete two-column proof is shown

Answer Choices:
- - - -
What is the missing statement in step 3

More info is in the picture
Thank you for any help!

Answers

The angles ∠KTW and ∠RWT are congruent with each other and WK = TR. Then the correct option is C.

What is the triangle?

The polygonal shape of a triangle has a number of sides and three independent variables. Angles in the triangle add up to 180°.

In isosceles trapezoid KRWT, the trianngles are ΔKTW and ΔRWT are formed.

In trianngles ΔKTW and ΔRWT, then we have

KW = TR        {Given}

∠W = ∠T       {Isosceles angles}

WT = WT       {Common side}

Then the trianngles ΔKTW and ΔRWT are congruent to each other.

The angles ∠KTW and ∠RWT are congruent with each other and WK = TR. Then the correct option is C.

More about the triangle link is given below.

https://brainly.com/question/25813512

#SPJ1

PLEASE HELP MISSING
WORK

Jonathan wants to save up enough money so that he can buy a new sports equipment set that includes a
football, baseball, soccer ball, and basketball. This complete boxed set costs $85.50. Jonathan has $22.00 he
saved from his birthday. In order to make more money, he plans to wash neighbors' windows. He plans to
charge $4 for each window he washes
Part A
Write and solve an inequality that represents the number of windows Jonathan can wash in order to save at
least the minimum amount he needs to buy the boxed set.
Inequality:_______


PART B
GRAPH THE SOLUTION ON A NUMBER LINE
Explain the meaning of your inequality from Part A based on the context of the problem

Answers

The inequality that shows the least amount he needs is  4x + 22 ≥ 85.50 which shows he needs at least $15.875

What is the inequality that show the least amount he needs to buy boxed set

The Inequality that will represent this problem is 4x + 22 ≥ 85.50, where x is the number of windows Jonathan washes.

To solve the inequality, we'll start by subtracting 22 from both sides:

4x ≥ 63.50

And finally, dividing both sides by 4:

x ≥ 15.875

So, Jonathan needs to wash at least 15 windows in order to save up enough money to buy the boxed set.

b.

The graph on the number line showing x ≥ 15.875 is attached below

Learn more on graph of inequality here;

https://brainly.com/question/28511630

#SPJ1

PLS HELPP

louise has planted 96 shrubs. the garden centre guaranteed her that at least 7/8 of the shrubs would survive. What is the minimum number od shrubs that should survive?

Answers

Answer:

84

What is a fraction?

A fraction is a fragment of a whole number, used to define parts of a whole. The whole can be a whole object, or many different objects. The number at the top of the line is called the numerator, whereas the bottom is called the denominator.

We can first convert 96 into a fraction.

96 = [tex]\frac{96}{1}[/tex]

Now that we know this, we can solve for the number of shrubs. If we know that at least [tex]\frac{7}{8}[/tex] of the shrubs should survive, we can use an equation that looks like this:

([tex]\frac{7}{8}[/tex]) × ([tex]\frac{96}{1}[/tex]) =?

Solving the equation:

([tex]\frac{7}{8}[/tex]) × ([tex]\frac{96}{1}[/tex]) = 84

Therefore, the minimum number of shrubs that should survive is 84.

The American flag has 50 stars, 7 red stripes, and 6 white stripes. Write a ratio to represent the ratio of stars to red stripes.

Answers

The ratio of stars to red stripes is 50 to 7

How to write a ratio to represent the ratio of stars to red stripes.

from the question, we have the following parameters that can be used in our computation:

Number of stars = 50

Red stripes = 7

White stripes = 6

Using the above as a guide, we have the following:

Ratio = Stars : Red stripes

substitute the known values in the above equation, so, we have the following representation

Stars : Red stripes = 50 : 7

Hence, the ratio is 50 : 7

Read more about ratio at

https://brainly.com/question/12024093

#SPJ1

Someone tell me that if you have 5 apples and the next day you got 10 times as many apples you had. Use the Property "Distributive Property of Multiplication"

Answers

Answer:

10x = 5 + 5 + 5 + 5 + 5 + 5 + 5 + 5 + 5 + 5

Step-by-step explanation:

Distributive property of multiplication says every element within every pair of elements is added and multiplied to each other. So the value of 10 times that of 5 is 50.

How do you solve this equation 4(x+3)= -8

Answers

Answer:

x= -5

Step-by-step explanation:

4(x + 3) = -8

4x + 12 = -8

4x = -20

x = -5

For time 0≤t≤10 , water is flowing into a small tub at a rate given by the function F defined by F(t)=arctan(π/2−t/10). For time 5≤t≤10 , water is leaking from the tub at a rate given by the function L defined by L(t)=0. 03(20t−t^2−75). Both F(t) and L(t) are measured in cubic feet per minute, and t is measured in minutes. The volume of water in the tub, in cubic feet, at time t minutes is given by W(t).


(a) At time t=3 , there are 2. 5 cubic feet of water in the tub. Write an equation for the locally linear approximation of W at t=3 , and use it to approximate the volume of water in the tub at time t=3. 5.

(b) Find W′′(8). Using correct units, interpret the meaning of W′′(8) in the context of the problem.

(c) Is there a time t , for 5
(d) The tub is in the shape of a rectangular box that is 0. 5 foot wide, 4 feet long, and 3 feet deep. What is the rate of change of the depth of the water in the tub at time t=6 ?

Answers

(a) W(3.5) ≈ 2.5 + 0.483(0.5) ≈ 2.7625 cubic feet.

(b) W''(8) ≈ -0.0397 - (-0.6) ≈ 0.5603 cubic feet per minute per minute.

(c) We can solve for t numerically using a graphing calculator or other numerical method. One possible solution is t ≈ 5

What is the quadratic equation?

The quadratic equation is a formula used to solve quadratic equations of the form ax² + bx + c = 0, where a, b, and c are constants and x is the variable.

(a) To find the locally linear approximation of W at t=3, we first need to find W(3) and W'(3).

We know that at time t=3, the volume of water in the tub is 2.5 cubic feet. Therefore, W(3) = 2.5.

To find W'(3), we need to use the fact that the rate of change of the volume of water in the tub is equal to the rate of water flowing in minus the rate of water leaking out.

So, W'(t) = F(t) - L(t).

At time t=3, we have F(3) = arctan(π/2 - 3/10) ≈ 1.383 cubic feet per minute, and L(3) = 0.03(20*3 - 3² - 75) = 0.9 cubic feet per minute. Therefore, W'(3) = 1.383 - 0.9 = 0.483 cubic feet per minute.

The locally linear approximation of W at t=3 is given by:

W(t) ≈ W(3) + W'(3)(t-3)

W(t) ≈ 2.5 + 0.483(t-3)

To approximate the volume of water in the tub at time t=3.5, we plug in t=3.5 into the above equation:

W(3.5) ≈ 2.5 + 0.483(0.5) ≈ 2.7625 cubic feet.

(b) To find W''(8), we first need to find an expression for W'(t). We know that:

W'(t) = F(t) - L(t)

Taking the derivative with respect to t:

W''(t) = F'(t) - L'(t)

We know that F(t) = arctan(π/2 - t/10), so F'(t) = -1/(10π/4 - t²/100 + t/5²) * (1/5), where we used the chain rule and the derivative of arctan(x) = 1/(1+x²). Evaluating this expression at t=8, we get F'(8) ≈ -0.0397 cubic feet per minute.

We also know that L(t) = 0.03(20t - t²- 75), so L'(t) = 0.03(20 - 2t). Evaluating this expression at t=8, we get L'(8) = -0.6 cubic feet per minute.

Therefore, W''(8) ≈ -0.0397 - (-0.6) ≈ 0.5603 cubic feet per minute per minute.

Interpreting the meaning of W''(8), we see that it represents the rate of change of the rate of change of the volume of water in the tub at time t=8. Specifically, W''(8) tells us how quickly the volume of water in the tub is changing with respect to time at time t=8 is changing.

(c) Yes, there is a time t for which the volume of water in the tub is neither increasing nor decreasing. This occurs when the rate of water flowing into the tub is equal to the rate of water leaking out. That is, we need to find a value of t such that F(t) = L(t).

We can solve for t numerically using a graphing calculator or other numerical method. One possible solution is t ≈ 5

To learn more about quadratic equation, Visit

https://brainly.com/question/1214333

#SPJ1

Out of 1000 students who appeared for C.A. Intermediate Examination, 750 failed in Math, 600 failed in Accounts and 600 failed in Costing, 450 failed in both Math & Accounts, 400 failed in both Math & Costing, 150 failed in both Accounts & Costing. The Students who failed in all the three Subjects were 75. Prove that the above data is not correct.

Answers

Yes, the data provided is not correct. This can be proven using the principle of inclusion-exclusion.

According to the given data, the total number of students who failed in Math is 750, the total number of students who failed in Accounts is 600, and the total number of students who failed in Costing is 600.

However, if we apply the principle of inclusion-exclusion, the total number of students who failed in at least one of the three subjects should be equal to the sum of the number of students who failed in each subject, minus the number of students who failed in two subjects, plus the number of students who failed in all three subjects.

Therefore, using this principle, we have:

750 + 600 + 600 - 450 - 400 + 75 = 975

This result shows that the number of students who failed in at least one of the three subjects is 975, which is greater than the total number of students who appeared for the examination (1000), which is not possible.

Therefore, the given data is not correct.

Find the slope of the line on the graph. Reduce all fractional answers to lowest terms.



-1/3
-3
-1

Answers

The slope of the line on the graph is equal to: A. -1/3.

How to calculate the slope of a line?

In Mathematics, the slope of any straight line can be determined by using this mathematical equation;

Slope (m) = (Change in y-axis, Δy)/(Change in x-axis, Δx)

Slope (m) = rise/run

Based on the information provided, we can logically deduce the following data points on the line:

Points on x-axis = (-2, 4).

Points on y-axis = (2, 0).

Substituting the given data points into the slope formula, we have the following;

Slope, m = (0 - 2)/(4 + 2)

Slope, m = -2/6

Slope, m = -1/3.

Read more on slope here: brainly.com/question/3493733

#SPJ1

Which of the following relations is a function?

Answers

Answer:

b

Step-by-step explanation:

make sure the y axis doesnt repeat

.. who. staple. food of the Vedic Aryan was ?

A) Barley and rice. B) MMilk

Answers

Answer: b

Step-by-step explanation:  ias  chinadu  

B because that's the VA which would be intellectually specialist.

Graph the linear inequality.
x < 2

Answers

There you go as shoes on the graph

Find the equation of the line with slope 7 which goes through the point (−2,−9).

Answers

[tex](\stackrel{x_1}{-2}~,~\stackrel{y_1}{-9})\hspace{10em} \stackrel{slope}{m} ~=~ 7 \\\\\\ \begin{array}{|c|ll} \cline{1-1} \textit{point-slope form}\\ \cline{1-1} \\ y-y_1=m(x-x_1) \\\\ \cline{1-1} \end{array}\implies y-\stackrel{y_1}{(-9)}=\stackrel{m}{ 7}(x-\stackrel{x_1}{(-2)}) \implies y +9= 7 (x +2) \\\\\\ y+9=7x+14\implies {\Large \begin{array}{llll} y=7x+5 \end{array}}[/tex]

Sherri saves nickels and dimes in a coin purse for her daughter. The total value of the coins in the purse is $0.95. The
number of nickels is 2 less than 5 times the number of dimes. How many nickels and how many dimes are in the coin
purse?

Answers

Let's represent the number of dimes in the coin purse as d. Then, the number of nickels in the coin purse can be represented as 5d - 2.

The total value of the coins can be expressed as:

0.05 * (5d - 2) + 0.10 * d = 0.95

Expanding and simplifying the equation, we get:

0.05 * 5d - 0.05 * 2 + 0.10 * d = 0.95

0.50d - 0.10 = 0.95

0.50d = 1.05

d = 2.10

Since the number of dimes must be a whole number, we round d down to 2. So, there are 2 dimes in the coin purse and 5 * 2 - 2 = 6 nickels in the coin purse.

PLEASE HELP WITH CHART:

Answers

You can fill in the rest of the table as shown below.

How to fill in the rest of the table?

The derivative of a function is a concept from calculus that measures the rate of change of a function at a specific point. It provides information about how the value of the function changes as its input changes.

Use the following differentiation rules to fill the table:

d/dx(f(x)·g(x)) = f(x)·g'(x) + f'(x)·g(x)         (product rule)

d/dx(f(x)/g(x)) = [ g(x)·f'(x) - f(x)·g'(x) ] / [(g(x))²]   (Quotient rule)

d/dx(g(x)/f(x)) = [ f(x)·g'(x) - g(x)·f'(x) ] / [(f(x))²]     (Quotient rule)

x    f(x)   f'(x)   g(x)  g'(x)    d/dx(f(x)·g(x))     d/dx(f(x)/g(x))    d/dx(g(x)/f(x))

0    3       -2    -4     3            17                      -1/16                    1/9

1     2       -1      1      0            -1                         -1                      1/4

2    4        2     3      1            10                       2/9                    -1/8

Learn more about derivatives of a function on:

https://brainly.com/question/25081524

#SPJ1

Given one form of a sequence, write the other form an= an-1 -4

Answers

In the algebra on of the form of sequence is arithmetic progression or sequence which is described below.

What is an arithmetic sequence?

In the arithmetic sequence  the common difference between any two consecutive terms remains constant .

In this sequence we can get the next term by adding one fixed value which is known as a common difference. It is also known as arithmetic progression(AP).

Formula for any nth term in AP,

T = a + nd

d = T₂-T₁

Where, a is first term , n is nth term and d is common difference

If aₙ = aₙ₋₁  -4

where aₙ is a nth term and aₙ₋₁ is a (n-1)th term

Now,

a + nd  = a + (n-1-1)d - 4

nd   = (n - 2)d - 4

nd = nd -2d - 4

2d = -4

d = -2

So, aₙ = a -2n

aₙ₋₁ = a -2(n - 2)

To know more about arithmetic sequence check:

https://brainly.com/question/18109692

#SPJ1

The inside diameter of a randomly selected piston ring is a random variable with mean value 11 cm and standard deviation 0.02 cm. Suppose the distribution of the diameter is normal. (Round your answers to four decimal places.)
(a) Calculate P(10.99 ? X ? 11.01) when n = 16.
(b) How likely is it that the sample mean diameter exceeds 11.01 when n = 25?

Answers

a) P(10.99 ≤ X ≤ 11.01) = 0.9544

b) Probability that the sample mean diameter exceeds 11.01 when n = 25 is 0.0062.

We are given the following in the question:

Mean, μ = 11 cm

Standard Deviation, σ = 0.02 cm

We are given that the distribution of diameter is a bell-shaped distribution that is a normal distribution.

Formula: z = (x - μ)/σ

a) P(10.99 ≤ X ≤ 11.01 when n = 16)

Standard error due to sampling = σ/n = 0.02/√16 = 0.005

P(10.99 ≤ X ≤ 11.01) = P((10.99 - 11)/0.005 ≤ z ≤ (11.01 - 11)/0.005)

                              = P(-2 ≤ z ≤ 2) = 0.9772-0.0228

                              = 0.9544 = 95.44%

b) P(sample mean diameter exceeds 11.01 when n = 25)

Standard error due to sampling =  σ/n = 0.02/√25 = 0.004

P(x > 11.01) = P(z > (11.01 - 11)/0.004) = P(z > 2.5)

                 = 1 - P(z ≤ 1) = 1 - 0.9938 = 0.0062 = 0.62%

To learn more about probability distribution here:

brainly.com/question/14210034

#SPJ4

Other Questions
this month, the roasting department of java coffee company completed and transferred out 75,000 one-pound bags of coffee. the department also had 25,000 units that were 80% complete with respect to conversion and no beginning work in process inventory. how many equivalent units of production for conversion does the department have for the month? 95,000 equivalent units 80,000 equivalent units 100,000 equivalent units 85,000 equivalent units PLSSS HELP!!! due at 8pmoriginal poem about color. Use at least three of these devices (repetition, rhyme, rhythm, personification, simile, metaphor, hyperbole) Whats an issue in the broader world that youve become passionate about? What experiences made you become passionate about it?PLEASE THIS IS DUE TODAY AND I REALLY NEED HELP. Please be specific and explain. GIVING OUT BRAINLIEST. How would administering a non-hydrolyzable form of GTP to cell affect its G protein-coupled signal transduction pathways? a. The pathways would lose their specificity of response b. The pathways would become hypersensitive to stimuli. c. The pathways would respond in unpredictable ways.d. The pathways would not turn off.e. The pathways would become nonresponsive to stimuli. The effects of a catalyst on a chemical reaction is to react with the product, effectively removing it and shifting the equilibrium to the right.(a) True(b) False different substances have different ____________ , or abilities to reflect light. which has the thickest filament a high resistance bulb or a low resistance bulb? how did most forced laborers or workers die during the holocaust? Five people will share a roll of ribbon equally. The roll has 32 feet of ribbon. How much ribbon, in feet, will each person get?A. 5/32B. 1/32C. 6 2/5D. 1/5 (27) Pendulum A has a 2 kg mass attached to a 1-meter length string. Pendulum B has a 4 kg mass attached to a 0.5-meter length string. What is the frequency of each string? Does the longer or shorter string have a higherfrequency?The 1 meter long stringb. The 0.5 meter long stringc. They are the same(28) A pendulum has a period of 8 seconds. What is the length and frequency of the string?a. 0.13 Hz and 16.21 metersc. 0.79 Hz and 640 metersb. 16.21 Hz and 0.13 metersd. 0.13 Hz and 1579.13 metersa. The accompanying table shows the results of a survey in which 250 mail and 250 female workers ages 25 to 64 or asked if they contribute to retirement savings plan at work find the probability that a randomly selected worker contribute to a retirement savings plan at work given the worker is male How many grams of CO2 and H2O are produced from the combustion of 220. g of propane (C3H8)? C3H8(g) + 5O2(g) 3CO2(g) + 4H2O(g) Which component in the circuit would most likely have the lowest resistance?A buzzerA capacitorA light bulbA wire a student stands at the edge of a cliff and throws a stone horizontally over the edge with a speed of vi 5 18.0 m/s. the cliff is h 5 50.0 m above a body of water as shown in figure p4.13. (a) what are the coordinates of the initial position of the stone? (b) what are the components of the initial velocity of the stone? (c) what is the appropriate analysis model for the vertical motion of the stone? (d) what is the appropriate analysis model for the horizontal motion of the stone? (e) write symbolic equations for the x and y components of the velocity of the stone as a function of time. (f) write symbolic equations for the position of the stone as a function of time. (g) how long after being released does the stone strike the water below the cliff? (h) with what speed and angle of impact does the stone land? The triangles are similar.What is the value of x?See the attachment for the problem to be answered. Required For each of the following independent events, identify the account that would be debited and the account that would be credited. The accounts for the first event are identified as an example. Event Account DebitedAccount Credited Cash a. Received cash by issuing common stock b. Received cash for services to be performed in the future. C. | Paid salaries payable. d. Provided services on account. e. Paid cash for operating expenses. f. Purchased supplies on account 9 Recognized revenue for services completed. Cash had been collected in Event b. h. Recognized accrued salaries expense Common stock Recognized expense for supplies used during the period j Performed services for cash. k. Paid accounts payable l. Received cash in payment of accounts receivable m. Paid a cash dividend to the stockholders Cleopatra and her brother, Ptolemy, each have cylindrical water clocks (so they drain uniformly), but their clocks have different heights and diameters. Cleopatras clock drains from full down to empty in 4 hours and Ptolemys does so in 5 hours. After two hours of draining, the water height in both clocks is the same. What fraction of the height of Cleopatras clock is the height of Ptolemys? After each has been draining for three hours, what fraction of the water height of Ptolemys clock is equal to the water height of Cleopatras? type your response on the box cell theory states that all living things contain one or more cells who do you think cell theory meets the definition of a scientific theory do you think it should be a scientific law ? explain your response The equation 7(2W + 3) = 14W + 20 has no solutions. Change one term in the original equation to create a new equation with infinitely many solutions. Help with these questions pls?